Skip to main content

Innovation Competition and Strategy

  • Chapter
  • First Online:
Book cover Optimal Control Applications for Operations Strategy
  • 845 Accesses

Abstract

Innovation enables the firm to develop and improve strong competitive advantage in the market. However, innovation doesn’t arise in a vacuum. Competition stirs innovation. That’s why I focus on innovation competition in the context of value chain. Innovation competition is a crucial factor in operations strategy. In this chapter, I first show how a firm’s learning capability affects its innovation competition and then explore the conditions under which competing firms are willing to collaborate to build a common infrastructure as public goods .

The original version of this chapter was revised. The erratum to this chapter is available at 10.1007/978-981-10-3599-9_6

This is a preview of subscription content, log in via an institution to check access.

Access this chapter

Chapter
USD 29.95
Price excludes VAT (USA)
  • Available as PDF
  • Read on any device
  • Instant download
  • Own it forever
eBook
USD 99.00
Price excludes VAT (USA)
  • Available as EPUB and PDF
  • Read on any device
  • Instant download
  • Own it forever
Softcover Book
USD 129.99
Price excludes VAT (USA)
  • Compact, lightweight edition
  • Dispatched in 3 to 5 business days
  • Free shipping worldwide - see info
Hardcover Book
USD 129.99
Price excludes VAT (USA)
  • Durable hardcover edition
  • Dispatched in 3 to 5 business days
  • Free shipping worldwide - see info

Tax calculation will be finalised at checkout

Purchases are for personal use only

Institutional subscriptions

Author information

Authors and Affiliations

Authors

Corresponding author

Correspondence to Bowon Kim .

Appendices

Appendix 1: An Example of Differential Games Problem

In this section, we demonstrate how to obtain a feedback solution for a differential games problem, utilizing the HJB equation .

1.1 Feedback Nash Equilibrium

Consider the general objective function as follows:

$$V^{i} = \int_{0}^{T} {\left( {\left( {a_{i} x_{i} - a_{j} x_{j} } \right) - e_{i} u_{i}^{2} + e_{j} u_{j}^{2} } \right)dt + H\left( {a_{i} x_{i} \left( T \right) - a_{j} x_{j} \left( T \right)} \right),\,i \ne j \,{\text{and}}\, i,j = 1,2}$$
(25)

More specifically, Player 1’s decision problem writes:

$$Maximize \int_{0}^{T} {\left( {\left( {a_{1} x_{1} - a_{2} x_{2} } \right) - e_{1} u_{1}^{2} + e_{2} u_{2}^{2} } \right)dt + H\left( {a_{1} x_{1} \left( T \right) - a_{2} x_{2} \left( T \right)} \right)}$$
(26)
$$\begin{aligned} Subject\,to\,x_{1}^{'} & = b_{1} u_{1} \\ x_{2}^{'} & = b_{2} u_{2} \\ x_{1} \left( 0 \right) & = x_{2} \left( 0 \right) = 0\,given. \\ \end{aligned}$$

Similarly, we define the decision problem for Player 2 as

$$Maximize \int_{0}^{T} {\left( {\left( { - a_{1} x_{1} + a_{2} x_{2} } \right) + e_{1} u_{1}^{2} - e_{2} u_{2}^{2} } \right)dt + H\left( { - a_{1} x_{1} \left( T \right) + a_{2} x_{2} \left( T \right)} \right)}$$
(27)
$$\begin{aligned} Subject\,to\,x_{1}^{'} & = b_{1} u_{1} \\ x_{2}^{'} & = b_{2} u_{2} \\ x_{1} \left( 0 \right) & = x_{2} \left( 0 \right) = 0\,given. \\ \end{aligned}$$

Then, we solve the problem as follows.

First, define the HJB equations for the players:

$$- J_{t}^{1} = max\left( {\left( {a_{1} x_{1} - a_{2} x_{2} } \right) - e_{1} u_{1}^{2} + e_{2} \bar{u}_{2}^{2} + J_{{x_{1} }}^{1} \left( {b_{1} u_{1} } \right) + J_{{x_{2} }}^{1} \left( {b_{2} \bar{u}_{2} } \right)} \right)$$
(28)
$$- J_{t}^{2} = max\left( {\left( { - a_{1} x_{1} + a_{2} x_{2} } \right) + e_{1} \bar{u}_{1}^{2} - e_{2} u_{2}^{2} + J_{{x_{1} }}^{2} \left( {b_{1} \bar{u}_{1} } \right) + J_{{x_{2} }}^{2} \left( {b_{2} u_{2} } \right)} \right)$$
(29)

From (28) and (29), obtain the optimal controls:

$$- 2e_{1} u_{1} + b_{1} J_{{x_{1} }}^{1} = 0\quad \therefore\,u_{1}^{*} = \frac{{b_{1} }}{{2e_{1} }}J_{{x_{1} }}^{1}$$
(30)
$$- 2e_{2} u_{2} + b_{2} J_{{x_{2} }}^{2} = 0\quad \therefore\,u_{2}^{*} = \frac{{b_{2} }}{{2e_{2} }}J_{{x_{2} }}^{2}$$
(31)

Putting (30) and (31) in (28), we get:

$$\begin{aligned} - J_{t}^{1} & = \left( {a_{1} x_{1} - a_{2} x_{2} } \right) - e_{1} \left( {\frac{{b_{1}^{2} }}{{4e_{1}^{2} }}\left( {J_{{x_{1} }}^{1} } \right)^{2} } \right) + e_{2} \left( {\frac{{b_{2}^{2} }}{{4e_{2}^{2} }}\left( {J_{{x_{2} }}^{2} } \right)^{2} } \right) + J_{{x_{1} }}^{1} b_{1} \left( {\frac{{b_{1} }}{{2e_{1} }}J_{{x_{1} }}^{1} } \right) + J_{{x_{2} }}^{1} b_{2} \left( {\frac{{b_{2} }}{{2e_{2} }}J_{{x_{2} }}^{2} } \right) \\ & = \left( {a_{1} x_{1} - a_{2} x_{2} } \right) + \frac{{b_{1}^{2} }}{{4e_{1} }}\left( {J_{{x_{1} }}^{1} } \right)^{2} + \frac{{b_{2}^{2} }}{{4e_{2} }}\left( {J_{{x_{2} }}^{2} } \right)^{2} + \frac{{b_{2}^{2} }}{{2e_{2} }}\left( {J_{{x_{2} }}^{1} J_{{x_{2} }}^{2} } \right) \\ \end{aligned}$$
(32)

From the structure of (25), we suggest

$$J^{i} = K\left( t \right)\left( {a_{i} x_{i} - a_{j} x_{j} } \right) + Q\left( t \right),\,i \ne j\,\, {\text{and}}\,\, i,j = 1, 2.$$
(33)

From (33), we obtain from (34) to (37):

$$J_{t}^{1} = K\left( t \right)^{'} \left( {a_{1} x_{1} - a_{2} x_{2} } \right) + Q\left( t \right)^{'}$$
(34)
$$J_{{x_{1} }}^{1} = a_{1} K\left( t \right)$$
(35)
$$J_{{x_{2} }}^{1} = - a_{2} K\left( t \right)$$
(36)
$$J_{{x_{2} }}^{2} = a_{2} K\left( t \right)$$
(37)

By plugging (34)–(37) into (38), we have (39):

$$- J_{t}^{1} = \left( {a_{1} x_{1} - a_{2} x_{2} } \right) + \frac{{b_{1}^{2} }}{{4e_{1} }}\left( {J_{{x_{1} }}^{1} } \right)^{2} + \frac{{b_{2}^{2} }}{{4e_{2} }}\left( {J_{{x_{2} }}^{2} } \right)^{2} + \frac{{b_{2}^{2} }}{{2e_{2} }}\left( {J_{{x_{2} }}^{1} J_{{x_{2} }}^{2} } \right)$$
(38)
$$\begin{aligned} - K\left( t \right)^{'} \left( {a_{1} x_{1} - a_{2} x_{2} } \right) - Q\left( t \right)^{'} & = \left( {a_{1} x_{1} - a_{2} x_{2} } \right) + \frac{{b_{1}^{2} }}{{4e_{1} }}\left( {a_{1} K\left( t \right)} \right)^{2} + \frac{{b_{2}^{2} }}{{4e_{2} }}\left( {a_{2} K\left( t \right)} \right)^{2} \\ & \quad +\,\frac{{b_{2}^{2} }}{{2e_{2} }}\left( { - a_{2} K\left( t \right)} \right)\left( {a_{2} K\left( t \right)} \right) \\ \end{aligned}$$
(39)

We rearrange (39) to get (40):

$$\therefore\,\left[ {\frac{{K\left( t \right)^{2} }}{4}\left( {\frac{{a_{1}^{2} b_{1}^{2} }}{{e_{1} }} - \frac{{a_{2}^{2} b_{2}^{2} }}{{e_{2} }}} \right) + Q\left( t \right)^{'} } \right] + \left( {a_{1} x_{1} - a_{2} x_{2} } \right)\left( {1 + K\left( t \right)^{'} } \right) = 0$$
(40)

To make (40) valid, we have to have:

$$1 + K\left( t \right)^{'} = 0\,{\text{and}}\,\frac{{K\left( t \right)^{2} }}{4}\left( {\frac{{a_{1}^{2} b_{1}^{2} }}{{e_{1} }} - \frac{{a_{2}^{2} b_{2}^{2} }}{{e_{2} }}} \right) + Q\left( t \right)^{'} = 0$$
(41)

The first part of (41) leads us to determine (42):

$$\begin{aligned} K\left( t \right)^{'} & = - 1,\,K(t) = - t + c_{1} ,\,{\text{where}}\,c_{1} \,{\text{is}}\,{\text{a}}\,{\text{constant}} \\ K\left( T \right) & = - T + c_{1} = H,\,{\text{therefore}}\,c_{1} = T + H \\ \end{aligned}$$
$$K(t) = - t + T + H$$
(42)

Using (42) in (35) and (37), we derive:

$$J_{{x_{1} }}^{1} = a_{1} K\left( t \right) = a_{1} \left( { - t + T + H} \right);\,J_{{x_{2} }}^{2} = a_{2} K\left( t \right) = a_{2} \left( { - t + T + H} \right)$$
(43)

With (43), we determine the optimal control variables as in (44) and (45):

$$u_{1}^{*} = \frac{{b_{1} }}{{2e_{1} }}J_{{x_{1} }}^{1} = \frac{{a_{1} b_{1} }}{{2e_{1} }}\left( { - t + T + H} \right)$$
(44)
$$u_{2}^{*} = \frac{{b_{2} }}{{2e_{2} }}J_{{x_{2} }}^{2} = \frac{{a_{2} b_{2} }}{{2e_{2} }}\left( { - t + T + H} \right)$$
(45)

Using (44) and (45), we derive the optimal state variables as in (46) and (47):

$$\begin{aligned} & x_{1}^{'} = b_{1} u_{1} = b_{1} \frac{{a_{1} b_{1} }}{{2e_{1} }}\left( { - t + T + H} \right) = \frac{{a_{1} b_{1}^{2} }}{{2e_{1} }}\left( { - t + T + H} \right) \\ & x_{1} \left( t \right) = - \frac{{a_{1} b_{1}^{2} }}{{4e_{1} }}t^{2} + \frac{{a_{1} b_{1}^{2} }}{{2e_{1} }}\left( {T + H} \right)t + k_{1} ,\,{\text{where}}\,k_{1} \,{\text{is}}\,{\text{a}}\,{\text{constant}} \\ \end{aligned}$$
$$x_{1} \left( 0 \right) = k_{1} = 0\,\therefore \,x_{1} \left( t \right) = - \frac{{a_{1} b_{1}^{2} }}{{4e_{1} }}t^{2} + \frac{{a_{1} b_{1}^{2} }}{{2e_{1} }}\left( {T + H} \right)t$$
(46)
$$\begin{aligned} & x_{2}^{'} = b_{2} u_{2} = b_{2} \frac{{a_{2} b_{2} }}{{2e_{2} }}\left( { - t + T + H} \right) = \frac{{a_{2} b_{2}^{2} }}{{2e_{2} }}\left( { - t + T + H} \right) \\ & x_{2} \left( t \right) = - \frac{{a_{2} b_{2}^{2} }}{{4e_{2} }}t^{2} + \frac{{a_{2} b_{2}^{2} }}{{2e_{2} }}\left( {T + H} \right)t + k_{2} ,\,{\text{where}}\,k_{2} \,{\text{is}}\,{\text{a}}\,{\text{constant}} \\ \end{aligned}$$
$$x_{2} \left( 0 \right) = k_{2} = 0\,\therefore \,x_{2} \left( t \right) = - \frac{{a_{2} b_{2}^{2} }}{{4e_{2} }}t^{2} + \frac{{a_{2} b_{2}^{2} }}{{2e_{2} }}\left( {T + H} \right)t$$
(47)

Using (41) and (42), we determine \(Q\left( t \right)\) as in (48):

$$\begin{aligned} Q\left( t \right)^{'} & = - \frac{{K\left( t \right)^{2} }}{4}\left( {\frac{{a_{1}^{2} b_{1}^{2} }}{{e_{1} }} - \frac{{a_{2}^{2} b_{2}^{2} }}{{e_{2} }}} \right) = - \frac{{\left( { - t + T + H} \right)^{2} }}{4}\left( {\frac{{a_{1}^{2} b_{1}^{2} }}{{e_{1} }} - \frac{{a_{2}^{2} b_{2}^{2} }}{{e_{2} }}} \right) \\ Q\left( t \right) & = \frac{{\left( { - t + T + H} \right)^{3} }}{12}\left( {\frac{{a_{1}^{2} b_{1}^{2} }}{{e_{1} }} - \frac{{a_{2}^{2} b_{2}^{2} }}{{e_{2} }}} \right) + c_{2} ,\,{\text{where}}\,c_{2} \,{\text{is}}\,{\text{a}}\,{\text{constant}} \\ Q\left( T \right) & = \frac{{H^{3} }}{12}\left( {\frac{{a_{1}^{2} b_{1}^{2} }}{{e_{1} }} - \frac{{a_{2}^{2} b_{2}^{2} }}{{e_{2} }}} \right) + c_{2} = 0 \\ \end{aligned}$$
$$\therefore\,Q\left( t \right) = \frac{{\left( { - t + T + H} \right)^{3} - H^{3} }}{12}\left( {\frac{{a_{1}^{2} b_{1}^{2} }}{{e_{1} }} - \frac{{a_{2}^{2} b_{2}^{2} }}{{e_{2} }}} \right)$$
(48)

Verification of Value Function: To verify the optimal solution, we need to confirm that we have indeed determined a right value function in (33). We do this in two ways, i.e., calculating the optimal total profit by utilizing the objective function in (26) and the value function in (33).

  1. A.

    Using the objective function in (26)

$$\begin{aligned} V^{1} & = \int_{0}^{T} {\left( {\left( {a_{1} x_{1} - a_{2} x_{2} } \right) - e_{1} u_{1}^{2} + e_{2} u_{2}^{2} } \right)dt + H\left( {a_{1} x_{1} \left( T \right) - a_{2} x_{2} \left( T \right)} \right)} \\ & = \int\limits_{0}^{T} {\left\{ {\left[ {a_{1} \left( { - \frac{{a_{1} b_{1}^{2} }}{{4e_{1} }}t^{2} + \frac{{a_{1} b_{1}^{2} }}{{2e_{1} }}\left( {T + H} \right)t} \right) - a_{2} \left( { - \frac{{a_{2} b_{2}^{2} }}{{4e_{2} }}t^{2} + \frac{{a_{2} b_{2}^{2} }}{{2e_{2} }}\left( {T + H} \right)t} \right)} \right]} \right. - e_{1} \left[ {\frac{{a_{1} b_{1} }}{{2e_{1} }}( - t} \right.} \\ &\quad \left. { + \left. {T + H)} \right]^{2} + e_{2} \left[ {\frac{{a_{2} b_{2} }}{{2e_{2} }}\left( { - t + T + H} \right)} \right]^{2} } \right\}dt \\ & \quad+ H\left\{ {a_{1} \left[ { - \frac{{a_{1} b_{1}^{2} }}{{4e_{1} }}T^{2} + \frac{{a_{1} b_{1}^{2} }}{{2e_{1} }}\left( {T + H} \right)T} \right]} \right. - \left. {a_{2} \left[ { - \frac{{a_{2} b_{2}^{2} }}{{4e_{2} }}T^{2} + \frac{{a_{2} b_{2}^{2} }}{{2e_{2} }}\left( {T + H} \right)T} \right]} \right\} \\ & = \frac{{T\left( {3H^{2} + 3HT + T^{2} } \right)}}{12}\left( {\frac{{a_{1}^{2} b_{1}^{2} }}{{e_{1} }} - \frac{{a_{2}^{2} b_{2}^{2} }}{{e_{2} }} } \right) = \frac{{\left( {T + H} \right)^{3} - H^{3} }}{12}\left( {\frac{{a_{1}^{2} b_{1}^{2} }}{{e_{1} }} - \frac{{a_{2}^{2} b_{2}^{2} }}{{e_{2} }}} \right). \\ \end{aligned}$$
(49)
  1. B.

    Using the value function in (33)

$$\begin{aligned} & J^{1} \left( {0, x_{1} \left( 0 \right), x_{2} \left( 0 \right)} \right) = K\left( 0 \right)\left( {a_{1} x_{1} \left( 0 \right) - a_{2} x_{2} \left( 0 \right)} \right) + Q\left( 0 \right) = K\left( 0 \right)\left( {a_{1} \times 0 - a_{2} \times 0} \right) \\ & + Q\left( 0 \right) = Q\left( 0 \right) = \frac{{\left( {T + H} \right)^{3} - H^{3} }}{12}\left( {\frac{{a_{1}^{2} b_{1}^{2} }}{{e_{1} }} - \frac{{a_{2}^{2} b_{2}^{2} }}{{e_{2} }}} \right) \\ \end{aligned}$$
(50)

Since (49) is the same as (50), we know that the value function is correct and we have obtained an optimal solution.

1.2 Open-Loop Nash Equilibrium

Now let’s find an open-loop Nash equilibrium solution for the differential games problem. Applying the maximum principle , we obtain optimal controls in (57) and (58) and also optimal state variables in (59) and (60).

$$H^{1} = \left( {a_{1} x_{1} - a_{2} x_{2} } \right) - e_{1} u_{1}^{2} + e_{2} u_{2}^{2} + \lambda_{1}^{1} b_{1} u_{1} + \lambda_{2}^{1} b_{2} u_{2}$$
(51)
$$H^{2} = \left( { - a_{1} x_{1} + a_{2} x_{2} } \right) + e_{1} u_{1}^{2} - e_{2} u_{2}^{2} + \lambda_{1}^{2} b_{1} u_{1} + \lambda_{2}^{2} b_{2} u_{2}$$
(52)
$$\lambda_{1}^{{1{\prime }}} = - H_{{x_{1} }}^{1} = - a_{1} ,\,\lambda_{1}^{1} \left( T \right) = a_{1} H\quad \therefore\,\lambda_{1}^{1} \left( t \right) = a_{1} \left( { - t + T + H} \right)$$
(53)
$$\lambda_{2}^{{1{\prime }}} = - H_{{x_{2} }}^{1} = a_{2} ,\,\lambda_{2}^{1} \left( T \right) = - a_{2} H\quad \therefore\, \lambda_{2}^{1} \left( t \right) = a_{2} \left( {t - T - H} \right)$$
(54)
$$\lambda_{1}^{{2{\prime }}} = - H_{{x_{1} }}^{2} = a_{1} ,\,\lambda_{1}^{2} \left( T \right) = - a_{1} H\quad \therefore\, \lambda_{1}^{2} \left( t \right) = a_{1} \left( {t - T - H} \right)$$
(55)
$$\lambda_{2}^{{2{\prime }}} = - H_{{x_{2} }}^{2} = - a_{2} ,\,\lambda_{2}^{2} \left( T \right) = a_{2} H\quad \therefore\, \lambda_{2}^{2} \left( t \right) = a_{2} \left( { - t + T + H} \right)$$
(56)
$$- 2e_{1} u_{1} + b_{1} \lambda_{1}^{1} = 0\quad \therefore\, u_{1}^{*} = \frac{{b_{1} }}{{2e_{1} }}\lambda_{1}^{1} = \frac{{a_{1} b_{1} }}{{2e_{1} }}\left( { - t + T + H} \right)$$
(57)
$$- 2e_{2} u_{2} + b_{2} \lambda_{2}^{2} = 0\quad \therefore\, u_{2}^{*} = \frac{{b_{2} }}{{2e_{2} }}\lambda_{2}^{2} = \frac{{a_{2} b_{2} }}{{2e_{2} }}\left( { - t + T + H} \right)$$
(58)
$$\begin{aligned} & x_{1}^{'} = b_{1} u_{1} = \frac{{a_{1} b_{1}^{2} }}{{2e_{1} }}\left( { - t + T + H} \right),\,x_{1} \left( 0 \right) = 0 \\ & \therefore \,x_{1} \left( t \right) = - \frac{{a_{1} b_{1}^{2} }}{{4e_{1} }}t^{2} + \frac{{a_{1} b_{1}^{2} }}{{2e_{1} }}\left( {T + H} \right)t \\ \end{aligned}$$
(59)
$$\begin{aligned} & x_{2}^{'} = b_{2} u_{2} = \frac{{a_{2} b_{2}^{2} }}{{2e_{2} }}\left( { - t + T + H} \right),\,x_{2} \left( 0 \right) = 0 \\ & \therefore \,x_{2} \left( t \right) = - \frac{{a_{2} b_{2}^{2} }}{{4e_{2} }}t^{2} + \frac{{a_{2} b_{2}^{2} }}{{2e_{2} }}\left( {T + H} \right)t \\ \end{aligned}$$
(60)

For this particular differential games problem, we confirm that the open-loop Nash equilibrium solution is equivalent to the feedback Nash equilibrium solution.

1.3 Numerical Example

In order to visualize the optimal dynamics, we conduct a numerical analysis, using the parameter values in Table 5.

Table 5 Parameter values

Figure 10 shows the optimal dynamics of the objective function and also those of the state variables. Figure 11 depicts the optimal dynamics of the control variables .

Fig. 10
figure 10

Optimal dynamics of objective function and state variables

Fig. 11
figure 11

Optimal dynamics of control variables

Appendix 2

1.1 Derivation of Theorem 1

Using the information in (19), we can reformulate each element of (17) as

  1. (i)

    \(J_{t}^{1} = \dot{h}_{1} \left( t \right)e^{{ - \lambda \left( {x_{1} + x_{2} } \right)}} + \dot{l}_{1} \left( t \right)e^{{ - \lambda x_{2} }}\)

  2. (ii)

    \(\begin{aligned} & \left( {J_{{x_{1} }}^{1} } \right)^{2} \frac{{3\alpha_{1}^{2} }}{{2K_{1} }}t^{{ - n_{1} }} e^{rt} e^{{\lambda \left( {x_{1} + x_{2} } \right)}} = \left( { - \lambda h_{1} \left( t \right)e^{{ - \lambda \left( {x_{1} + x_{2} } \right)}} } \right)^{2} \frac{{3\alpha_{1}^{2} }}{{2K_{1} }}t^{{ - n_{1} }} e^{rt} e^{{\lambda \left( {x_{1} + x_{2} } \right)}} \\ & = \lambda^{2} \frac{{3\alpha_{1}^{2} }}{{2K_{1} }}t^{{ - n_{1} }} e^{rt} e^{{\lambda \left( {x_{1} + x_{2} } \right)}} h_{1}^{2} \\ \end{aligned}\)

  3. (iii)

    \(\begin{aligned} & J_{{x_{2} }}^{1} J_{{x_{2} }}^{2} \frac{{\alpha_{2}^{2} }}{{K_{2} }}t^{{ - n_{2} }} e^{rt} e^{{\lambda \left( {x_{1} + x_{2} } \right)}} \\ & = \left( { - \lambda h_{1} \left( t \right)e^{{ - \lambda \left( {x_{1} + x_{2} } \right)}} - \lambda l_{1} \left( t \right)e^{{ - \lambda x_{2} }} } \right)\left( { - \lambda h_{2} \left( t \right)e^{{ - \lambda \left( {x_{1} + x_{2} } \right)}} } \right)\frac{{\alpha_{2}^{2} }}{{K_{2} }}t^{{ - n_{2} }} e^{rt} e^{{\lambda \left( {x_{1} + x_{2} } \right)}} \\ & = \lambda^{2} \frac{{\alpha_{2}^{2} }}{{K_{2} }}t^{{ - n_{2} }} e^{rt} e^{{ - \lambda \left( {x_{1} + x_{2} } \right)}} h_{1} h_{2} + \lambda^{2} \frac{{\alpha_{2}^{2} }}{{K_{2} }}t^{{ - n_{2} }} e^{rt} e^{{ - \lambda x_{2} }} l_{1} h_{2} \\ \end{aligned}\)

  4. (iv)

    \(\begin{aligned} & J_{{x_{2} }}^{2} \frac{{\alpha_{2}^{2} }}{{K_{2} }}R_{0} t^{{m - n_{2} }} e^{rt} \lambda \left( {e^{{\lambda x_{1} }} - 1} \right) = - \lambda h_{2} \left( t \right)e^{{ - \lambda \left( {x_{1} + x_{2} } \right)}} \frac{{\alpha_{2}^{2} }}{{K_{2} }}R_{0} t^{{m - n_{2} }} e^{rt} \lambda \left( {e^{{\lambda x_{1} }} - 1} \right) = \\ & - \lambda^{2} \frac{{\alpha_{2}^{2} }}{{K_{2} }}R_{0} t^{{m - n_{2} }} e^{rt} e^{{ - \lambda x_{2} }} h_{2} + \lambda^{2} \frac{{\alpha_{2}^{2} }}{{K_{2} }}R_{0} t^{{m - n_{2} }} e^{rt} e^{{ - \lambda \left( {x_{1} + x_{2} } \right)}} h_{2} \\ \end{aligned}\)

  5. (v)

    \(mR_{0} t^{m - 1} \left( {1 - e^{{ - \lambda x_{1} }} } \right)e^{{ - \lambda x_{2} }} = mR_{0} t^{m - 1} e^{{ - \lambda x_{2} }} - mR_{0} t^{m - 1} e^{{ - \lambda \left( {x_{1} + x_{2} } \right)}}\)

By putting (i)–(v) into (17) and rearranging, we obtain:

$$E_{1} \left( t \right)e^{{ - \lambda \left( {x_{1} + x_{2} } \right)}} + E_{2} \left( t \right)e^{{ - \lambda x_{2} }} = 0,$$
(61)

where \(E_{1} \left( t \right) = \dot{h}_{1} + \lambda^{2} e^{rt} \left( {\frac{{3\alpha_{1}^{2} }}{{2K_{1} }}t^{{ - n_{1} }} h_{1}^{2} + \frac{{\alpha_{2}^{2} }}{{K_{2} }}t^{{ - n_{2} }} h_{1} h_{2} } \right) + \lambda^{2} e^{rt} \frac{{\alpha_{2}^{2} }}{{K_{2} }}R_{0} t^{{m - n_{2} }} h_{2} + mR_{0} t^{m - 1}\)and

$$\begin{aligned} E_{2} \left( t \right) & = \dot{l}_{1} + \lambda^{2} e^{rt} \frac{{\alpha_{2}^{2} }}{{K_{2} }}t^{{ - n_{2} }} l_{1} h_{2} - \lambda^{2} e^{rt} \frac{{\alpha_{2}^{2} }}{{K_{2} }}R_{0} t^{{m - n_{2} }} h_{2} - mR_{0} t^{m - 1} \\ & = \dot{l}_{1} + \lambda^{2} e^{rt} \frac{{\alpha_{2}^{2} }}{{K_{2} }}t^{{ - n_{2} }} \left( {l_{1} - R_{0} t^{m} } \right)h_{2} - mR_{0} t^{m - 1} . \\ \end{aligned}$$

In order to satisfy (61), both \(E_{1} \left( t \right)\) and \(E_{2} \left( t \right)\) must be zero. In addition, we can confirm that \(l_{1} \left( t \right) = R_{0} t^{m}\) makes \(E_{2} \left( t \right) = 0\).

1.2 Derivation of Corollary 1

Following steps similar with those for Firm 1, we can propose

$$J^{2} \left( {t,x_{1} ,x_{2} } \right) = h_{2} \left( t \right)e^{{ - \lambda \left( {x_{1} + x_{2} } \right)}} + l_{2} \left( t \right)e^{{ - \lambda x_{1} }} .$$

Using (19) and rearranging appropriately, we have the following condition for optimality: \(G_{1} \left( t \right)e^{{ - \lambda \left( {x_{1} + x_{2} } \right)}} + G_{2} \left( t \right)e^{{ - \lambda x_{1} }} = 0\), where

$$\begin{aligned} G_{1} \left( t \right) & = \dot{h}_{2} + \lambda^{2} e^{rt} \left( {\frac{{3\alpha_{2}^{2} }}{{2K_{2} }}t^{{ - n_{2} }} h_{2}^{2} + \frac{{\alpha_{1}^{2} }}{{K_{1} }}t^{{ - n_{1} }} h_{1} h_{2} } \right) + \lambda^{2} e^{rt} \frac{{\alpha_{1}^{2} }}{{K_{1} }}R_{0} t^{{m - n_{1} }} h_{1} + mR_{0} t^{m - 1} \,{\text{and}} \\ G_{2} \left( t \right) & = \dot{l}_{2} + \lambda^{2} e^{rt} \frac{{\alpha_{1}^{2} }}{{K_{1} }}t^{{ - n_{1} }} l_{2} h_{1} - \lambda^{2} e^{rt} \frac{{\alpha_{1}^{2} }}{{K_{1} }}R_{0} t^{{m - n_{1} }} h_{1} - mR_{0} t^{m - 1} \\ & = \dot{l}_{2} + \lambda^{2} e^{rt} \frac{{\alpha_{1}^{2} }}{{K_{1} }}t^{{ - n_{1} }} \left( {l_{2} - R_{0} t^{m} } \right)h_{1} - mR_{0} t^{m - 1} . \\ \end{aligned}$$

Again, it must be that \(G_{1} \left( t \right) = G_{2} \left( t \right) = 0\). We can also show that \(l_{2} = R_{0} t^{m}\) makes \(G_{2} \left( t \right) = 0\).

1.3 Derivation of Theorem 3

Consider for \(i = 1\), using HJB equation for an infinity case:

$$\begin{aligned} & rJ = \max_{{u_{1} }} \left\{ {\left( {x_{1} + x_{2} } \right)\left[ {a_{i} - (x_{1} + x_{2} )} \right] + s_{i} x_{i} \left[ {b_{i} - x_{i} } \right] - \rho u_{i} - \frac{1}{2}u_{i}^{2} + J_{{x_{1} }} \left( {u_{1} - \delta x_{1} } \right)} \right. \\ & \quad + \left. {J_{{x_{2} }} (u_{2} - \delta x_{2} )} \right\} \\ \end{aligned}$$
(62)

Thus, we obtain: \(u_{1} = J_{{x_{1} }} - \rho ,\,u_{2} = J_{{x_{2} }}^{2} - \rho\).

We suggest the following value functions :

$$\begin{aligned} J & = \frac{a}{2}\left( {x_{1} + x_{2} } \right)^{2} + \beta_{1} \left( {x_{1} + x_{2} } \right) + \frac{\theta }{2}x_{1}^{2} + \phi_{1} x_{1} + \gamma_{1} \,{\text{for}}\,i = 1\,,\,{\text{and}} \\ J^{2} & = \frac{a}{2}\left( {x_{1} + x_{2} } \right)^{2} + \beta_{2} \left( {x_{1} + x_{2} } \right) + \frac{\theta }{2}x_{2}^{2} + \phi_{2} x_{2} + \gamma_{2} \,{\text{for}}\,i = 2. \\ \end{aligned}$$

Take appropriate partial differentiations and plug them into (62):

$$\begin{aligned} & J_{{x_{1} }} = \alpha \left( {x_{1} + x_{2} } \right) + \beta_{1} + \theta x_{1} + \phi_{1} \\ & J_{{x_{2} }} = \alpha \left( {x_{1} + x_{2} } \right) + \beta_{1} \\ & J_{{x_{2} }}^{2} = \alpha \left( {x_{1} + x_{2} } \right) + \beta_{2} + \theta x_{2} + \phi_{2} \\ & \therefore\, rJ = \left( {x_{1} + x_{2} } \right)\left[ {a_{1} - \left( {x_{1} + x_{2} } \right)} \right] + s_{1} x_{1} \left[ {b_{1} - x_{1} } \right] - \rho \left( {J_{{x_{1} }} - \rho } \right) - \frac{1}{2}\left( {J_{{x_{1} }} - \rho } \right)^{2} \\ & + J_{{x_{1} }} \left( {J_{{x_{1} }} - \rho - \delta x_{1} } \right) + J_{{x_{2} }} (J_{{x_{2} }}^{2} - \rho - \delta x_{2} ) \\ & \therefore\, rJ = \left( {x_{1} + x_{2} } \right)\left[ {a_{1} - \left( {x_{1} + x_{2} } \right)} \right] + s_{1} x_{1} \left[ {b_{1} - x_{1} } \right] + \frac{1}{2}\left( {J_{{x_{1} }} } \right)^{2} - \rho J_{{X_{1} }} - \delta x_{1} J_{{x_{1} }} + \frac{1}{2}\rho^{2} \\ & + J_{{x_{2} }} J_{{x_{2} }}^{2} - \rho J_{{x_{2} }} - \delta x_{2} J_{{x_{2} }} \\ \end{aligned}$$

For simplicity, we consider separate cases:

  1. (1)

    \(\begin{aligned} \frac{1}{2}\left( {J_{{x_{1} }} } \right)^{2} & = \frac{1}{2}\left\{ {\alpha^{2} \left( {x_{1} + x_{2} } \right)^{2} + \theta^{2} x_{1}^{2} + \left( {\beta_{1} + \phi_{1} } \right)^{2} + 2\alpha \theta x_{1} \left( {x_{1} + x_{2} } \right) + 2\alpha \left( {\beta_{1} } \right.} \right. \\ & \quad\quad\,\, \left. { + \phi_{1} )\left( {x_{1} + x_{2} } \right) + 2\theta \left( {\beta_{1} + \phi_{1} } \right)x_{1} } \right\} \\ \end{aligned}\)

  2. (2)

    \(\rho J_{{x_{1} }} = \alpha \rho \left( {x_{1} + x_{2} } \right) + \theta \rho x_{1} + \rho (\beta_{1} + \phi_{1} )\)

  3. (3)

    \(\delta x_{1} J_{{x_{1} }} = \alpha \delta x_{1} \left( {x_{1} + x_{2} } \right) + \theta \delta x_{1}^{2} + \delta \left( {\beta_{1} + \phi_{1} } \right)x_{1}\)

  4. (4)

    \(\begin{aligned} J_{{x_{2} }} J_{{x_{2} }}^{2} & = \alpha^{2} \left( {x_{1} + x_{2} } \right)^{2} + \alpha \theta x_{2} \left( {x_{1} + x_{2} } \right) + \alpha \left( {\beta_{2} + \phi_{2} } \right)\left( {x_{1} + x_{2} } \right) + \alpha \beta_{1} \left( {x_{1} + x_{2} } \right) \\ & \quad + \theta \beta_{1} x_{2} + \beta_{1} (\beta_{2} + \phi_{2} ) \\ \end{aligned}\)

  5. (5)

    \(\rho J_{{x_{2} }} = \alpha \rho \left( {x_{1} + x_{2} } \right) + \rho \beta_{1}\)

  6. (6)

    \(\delta x_{2} J_{{x_{2} }} = \alpha \delta x_{2} \left( {x_{1} + x_{2} } \right) + \delta \beta_{1} x_{2}\)

Now,

$$\begin{aligned} & \therefore\, - r\frac{\alpha }{2}\left( {x_{1} + x_{2} } \right)^{2} - r\beta_{1} \left( {x_{1} + x_{2} } \right) - r\frac{\theta }{2}x_{1}^{2} - r\phi_{1} x_{1} - r\gamma_{1} \\ & + a_{1} \left( {x_{1} + x_{2} } \right) - \left( {x_{1} + x_{2} } \right)^{2} + s_{1} b_{1} x_{1} - s_{1} x_{1}^{2} \\ & + \frac{1}{2}\alpha^{2} \left( {x_{1} + x_{2} } \right)^{2} + \frac{1}{2}\theta^{2} x_{1}^{2} + \frac{1}{2}\left( {\beta_{1} + \phi_{1} } \right)^{2} + \alpha \theta x_{1} (x_{1} + x_{2} ) \\ & + \alpha (\beta_{1} + \phi_{1} )(x_{1} + x_{2} ) + \theta (\beta_{1} + \phi_{1} )x_{1} \\ & - \alpha \rho \left( {x_{1} + x_{2} } \right) - \theta \rho x_{1} - \rho (\beta_{1} + \phi_{1} ) \\ & - \alpha \delta x_{1} \left( {x_{1} + x_{2} } \right) - \theta \delta x_{1}^{2} - \delta \left( {\beta_{1} + \phi_{1} } \right)x_{1} + \frac{1}{2}\rho^{2} \\ & +\upalpha^{2} \left( {x_{1} + x_{2} } \right)^{2} + a\theta x_{2} \left( {x_{1} + x_{2} } \right) + \alpha \left( {\beta_{2} + \phi_{2} } \right)\left( {x_{1} + x_{2} } \right) + \alpha \beta_{1} \left( {x_{1} + x_{2} } \right) + \theta \beta_{1} x_{2} \\ & + \beta_{1} \left( {\beta_{2} + \phi_{2} } \right) - \alpha \rho \left( {x_{1} + x_{2} } \right) - \rho \beta_{1} - \alpha \delta x_{2} \left( {x_{1} + x_{2} } \right) - \delta \beta_{1} x_{2} = 0 \\ \end{aligned}$$
$$\begin{aligned} & \therefore\, \left\{ { - r\frac{a}{2} - 1 + \frac{3}{2}\alpha^{2} } \right\}\left( {x_{1} + x_{2} } \right)^{2} \\ & + \left\{ { - r\beta_{1} + a_{1} + \alpha \left( {\beta_{1} + \phi_{1} } \right) - \alpha \rho + \alpha \left( {\beta_{2} + \phi_{2} } \right) + \alpha \beta_{1} - \alpha \rho } \right\}(x_{1} + x_{2} ) \\ & + \left\{ { - r\frac{\theta }{2} - s_{1} + \frac{1}{2}\theta^{2} - \theta \delta } \right\}x_{1}^{2} + \left\{ { - r\phi_{1} + s_{1} b_{1} + \theta \left( {\beta_{1} + \phi_{1} } \right) - \theta \rho - \delta (\beta_{1} + \phi_{1} )} \right\}x_{1} \\ & + \left\{ {\alpha \theta - \alpha \delta } \right\}x_{1} \left( {x_{1} + x_{2} } \right) + \left\{ {\alpha \theta - \alpha \delta } \right\}x_{2} \left( {x_{1} + x_{2} } \right) + \left\{ {\theta \beta_{1} - \delta \beta_{1} } \right\}x_{2} \\ & + \left\{ { - r\gamma_{1} + \frac{1}{2}\left( {\beta_{1} + \phi_{1} } \right)^{2} - \rho \left( {\beta_{1} + \phi_{1} } \right) + \frac{1}{2}\rho^{2} + \beta_{1} \left( {\beta_{2} + \phi_{2} } \right) - \rho \beta_{1} } \right\} = 0 \\ \end{aligned}$$
$$\therefore\, Ax_{1}^{2} + Bx_{1} x_{2} + Cx_{2}^{2} + Dx_{1} + Ex_{2} + F = 0$$

Therefore, we must have: A = B = C = D = E = F = 0, where

$$\begin{aligned} A & = \left\{ { - r\frac{\alpha }{2} + 1 + \frac{3}{2}\alpha^{2} } \right\} + \left\{ { - r\frac{\theta }{2} - s_{1} + \frac{1}{2}\theta^{2} - \theta \delta } \right\} + \left\{ {\alpha \theta - \alpha \delta } \right\} \\ B & = 2\left\{ { - r\frac{\alpha }{2} - 1 + \frac{3}{2}\alpha^{2} } \right\} + \left\{ {\alpha \theta - \alpha \delta } \right\} + \left\{ {\alpha \theta - \alpha \delta } \right\} \\ C & = \left\{ { - r\frac{\alpha }{2} - 1 + \frac{3}{2}\alpha^{2} } \right\} + \left\{ {\alpha \theta - \alpha \delta } \right\} \\ D & = \left\{ { - r\beta_{1} + a_{1} + \alpha \left( {2\beta_{1} + \beta_{2} + \phi_{1} + \phi_{2} - 2\rho } \right)} \right\} + \left\{ { - r\phi_{1} + s_{1} b_{1} + \left( {\theta - \delta } \right)\left( {\beta_{1} + \phi_{1} } \right) - \theta \rho } \right\} \\ E & = \left\{ { - r\beta_{1} + a_{1} + \alpha \left( {2\beta_{1} + \beta_{2} + \phi_{1} + \phi_{2} - 2\rho } \right)} \right\} + \left\{ {\theta \beta_{1} - \delta \beta_{1} } \right\} \\ F & = - r\gamma_{1} + \frac{1}{2}\left( {\beta_{1} + \phi_{1} } \right)^{2} - \rho \left( {\beta_{1} + \phi_{1} } \right) + \frac{1}{2}\rho^{2} + \beta_{1} \left( {\beta_{2} + \phi_{2} } \right) - \rho \beta_{1} . \\ \end{aligned}$$

From A = C = 0, it must be that \(\left\{ { - r\frac{\theta }{2} - s_{1} + \frac{1}{2}\theta^{2} - \theta \delta } \right\} = \frac{1}{2}\theta^{2} - \left( {\frac{r}{2} + \delta } \right)\theta - s_{1} = 0\).

Therefore, \(\theta = \frac{{(r + 2\delta ) \pm \sqrt {\left( {r + 2\delta } \right)^{2} + 8s_{1} } }}{2}\). We also need to impose \(\theta < 0\), thus \(\theta = \frac{{\left( {r + 2\delta } \right) - \sqrt {\left( {r + 2\delta } \right)^{2} + 8s_{1} } }}{2}.\)

From B = C = 0, \(\left\{ { - r\frac{\alpha }{2} - 1 + \frac{3}{2}\alpha^{2} } \right\} + \left\{ {\alpha \theta - \alpha \delta } \right\} = \frac{3}{2}\alpha^{2} - \left( {\frac{r}{2} + \delta - \theta } \right)\alpha - 1 = 0\), and \(3\alpha^{2} - \left( {r + 2\delta - 2\theta } \right)\alpha - 2 = 0\). Therefore, \(\upalpha = \frac{{\left( {r + 2\delta - 2\theta } \right) \pm \sqrt {\left( {r + 2\delta - 2\theta } \right)^{2} + 24} }}{6}\). Due to the condition \(\alpha < 0\), \(\alpha = \frac{{\left( {r + 2\delta - 2\theta } \right) - \sqrt {\left( {r + 2\delta - 2\theta } \right)^{2} + 24} }}{6}\).

Taking the similar steps for i = 2, we obtain the following:

$$\begin{aligned} D^{2} & = \left\{ { - r\beta_{2} + a_{2} + \alpha \left( {2\beta_{2} + \beta_{1} + \phi_{1} + \phi_{2} - 2\rho } \right)} \right\} + \left\{ { - r\phi_{2} + s_{2} b_{2} + \left( {\theta - \delta } \right)\left( {\beta_{2} + \phi_{2} } \right) - \theta \rho } \right\} \\ E^{2} & = \left\{ { - r\beta_{2} + a_{2} + \alpha \left( {2\beta_{2} + \beta_{1} + \phi_{1} + \phi_{2} - 2\rho } \right)} \right\} + \left\{ {\theta \beta_{2} - \delta \beta_{2} } \right\} \\ F^{2} & = - r\gamma_{2} + \frac{1}{2}\left( {\beta_{2} + \phi_{2} } \right)^{2} - \rho \left( {\beta_{2} + \phi_{2} } \right) + \frac{1}{2}\rho^{2} + \beta_{2} \left( {\beta_{1} + \phi_{1} } \right) - \rho \beta_{2} \\ \end{aligned}$$

Now, we have 4 equations, D, E, D 2, and \(E^{2}\), and 4 variables, \(\beta_{1}\), \(\beta_{2}\), \(\phi_{1}\), and \(\phi_{2}\), to determine.

$$K\left[ {\begin{array}{*{20}c} {\beta_{1} } \\ {\beta_{2} } \\ {\phi_{1} } \\ {\phi_{2} } \\ \end{array} } \right] = L,\,{\text{where}}\,K \equiv \left[ {\begin{array}{*{20}c} {2\alpha + \theta - r - \delta } & \alpha & {\alpha + \theta - r - \delta } & \alpha \\ {2\alpha + \theta - r - \delta } & \alpha & \alpha & \alpha \\ \alpha & {2\alpha + \theta - r - \delta } & \alpha & {\alpha + \theta - r - \delta } \\ \alpha & {2\alpha + \theta - r - \delta } & \alpha & \alpha \\ \end{array} } \right]$$

and \(L \equiv - \left[ {\begin{array}{*{20}c} {a_{1} - 2\alpha \rho + s_{1} b_{1} - \theta \rho } \\ {a_{1} - 2\alpha \rho } \\ {a_{2} - 2\alpha \rho + s_{2} b_{2} - \theta \rho } \\ {a_{2} - 2\alpha \rho } \\ \end{array} } \right]\)

$$\left. { \times \left[ {\left. {\begin{array}{*{20}c} {2\alpha + \theta - r - \delta } & \alpha & {\alpha + \theta - r - \delta } & \alpha \\ {2\alpha + \theta - r - \delta } & \alpha & \alpha & \alpha \\ \alpha & {2\alpha + \theta - r - \delta } & \alpha & {\alpha + \theta - r - \delta } \\ \alpha & {2\alpha + \theta - r - \delta } & \alpha & \alpha \\ \end{array} } \right|} \right.\begin{array}{*{20}c} {2\alpha \rho + \theta \rho - a_{1} - s_{1} b_{1} } \\ {2\alpha \rho - a_{1} } \\ {2\alpha \rho + \theta \rho - a_{2} - s_{2} b_{2} } \\ {2\alpha \rho - a_{2} } \\ \end{array} } \right]$$
$$\left. { \times \left[ {\left. {\begin{array}{*{20}c} {2\alpha + \theta - r - \delta } & \alpha & {\alpha + \theta - r - \delta } & \alpha \\ 0 & 0 & {r + \delta - \theta } & 0 \\ \alpha & {2\alpha + \theta - r - \delta } & \alpha & {\alpha + \theta - r - \delta } \\ 0 & 0 & 0 & {r + \delta - \theta } \\ \end{array} } \right|} \right.\begin{array}{*{20}c} {2\alpha \rho + \theta \rho - a_{1} - s_{1} b_{1} } \\ {s_{1} b_{1} - \theta \rho } \\ {2\alpha \rho + \theta \rho - a_{2} - s_{2} b_{2} } \\ {s_{2} b_{2} - \theta \rho } \\ \end{array} } \right]$$

\(\therefore\, \phi_{1} = \frac{{s_{1} b_{1} - \theta \rho }}{r + \delta - \theta }\) and \(\phi_{2} = \frac{{s_{2} b_{2} - \theta \rho }}{r + \delta - \theta }\)

$$\begin{aligned} & \left( {2\alpha + \theta - r - \delta } \right)\beta_{1} + \alpha \beta_{2} + \frac{{\left( {\alpha + \theta - r - \delta } \right)\left( {s_{1} b_{1} - \theta \rho } \right)}}{r + \delta - \theta } + \frac{{\alpha \left( {s_{2} b_{2} - \theta \rho } \right)}}{r + \delta - \theta } = 2\alpha \rho + \theta \rho - a_{1} - s_{1} b_{1} \\ & \alpha \beta_{1} + \left( {2\alpha + \theta - r - \delta } \right)\beta_{2} + \frac{{\alpha \left( {s_{1} b_{1} - \theta \rho } \right)}}{r + \delta - \theta } + \frac{{\left( {\alpha + \theta - r - \delta } \right)\left( {s_{2} b_{2} - \theta \rho } \right)}}{r + \delta - \theta } = 2\alpha \rho + \theta \rho - a_{2} - s_{2} b_{2} \\ \end{aligned}$$

Let’s define \(W_{i} = \frac{{\alpha \left( {2r\rho + 2\delta \rho - s_{1} b_{1} - s_{2} b_{2} } \right)}}{r + \delta - \theta } - a_{i}\).

Then, \(\left( {2\alpha + \theta - r - \delta } \right)\beta_{1} + \alpha \beta_{2} = W_{1}\) and \(\alpha \beta_{1} + \left( {2\alpha + \theta - r - \delta } \right)\beta_{2} = W_{2}\).

$$\begin{aligned} \therefore\, \beta_{1} & = \frac{2\alpha + \theta - r - \delta }{{\left( {2\alpha + \theta - r - \delta } \right)^{2} - \alpha^{2} }}W_{1} + \frac{ - \alpha }{{\left( {2\alpha + \theta - r - \delta } \right)^{2} - \alpha^{2} }}W_{2} \,{\text{and}} \\ \beta_{2} & = \frac{ - \alpha }{{\left( {2\alpha + \theta - r - \delta } \right)^{2} - \alpha^{2} }}W_{1} + \frac{2\alpha + \theta - r - \delta }{{\left( {2\alpha + \theta - r - \delta } \right)^{2} - \alpha^{2} }}W_{2} \\ \end{aligned}$$

Finally, from \(F,\,\gamma_{1} = \frac{1}{r}\left[ {\frac{1}{2}\left( {\beta_{1} + \phi_{1} } \right)^{2} - \rho \left( {\beta_{1} + \phi_{1} } \right) + \frac{1}{2}\rho^{2} + \beta_{1} \left( {\beta_{2} + \phi_{2} } \right) - \rho \beta_{1} } \right].\)

Likewise, from \(F^{2} ,\,\gamma_{2} = \frac{1}{r}\left[ {\frac{1}{2}\left( {\beta_{2} + \phi_{2} } \right)^{2} - \rho \left( {\beta_{2} + \phi_{2} } \right) + \frac{1}{2}\rho^{2} + \beta_{2} \left( {\beta_{1} + \phi_{1} } \right) - \rho \beta_{2} } \right]\).

$$\begin{aligned} & {\text{Now}},\,u_{1} = J_{{x_{1} }} - \rho = \alpha \left( {x_{1} + x_{2} } \right) + \theta x_{1} + \beta_{1} + \phi_{1} - \rho \,{\text{and}} \\ & u_{2} = J_{{x_{2} }}^{2} - \rho = \alpha \left( {x_{1} + x_{2} } \right) + \theta x_{2} + \beta_{2} + \phi_{2} - \rho . \\ \end{aligned}$$

In order to get exact expressions, we need to do the following:

$$\begin{aligned} \dot{x}_{1} &= u_{1} - \delta x_{1} = \alpha \left( {x_{1} + x_{2} } \right) + \left( {\theta - \delta } \right)x_{1} + \beta_{1} + \phi_{1} - \rho \\ & = \left( {\alpha + \theta - \delta } \right)x_{1} + \alpha x_{2} + \beta_{1}+ \phi_{1} - \rho \\ \end{aligned}$$
$$\begin{aligned} \dot{x}_{2} & = u_{2} - \delta x_{2} = \alpha \left( {x_{1} + x_{2} } \right) + \left( {\theta - \delta } \right)x_{2} + \beta_{2} + \phi_{2} - \rho \\ &= \alpha x_{1} + \left( {\alpha + \theta - \delta } \right)x_{2} + \beta_{2} + \phi_{2} - \rho . \\ \end{aligned}$$

Homogeneous equations:

$$\begin{aligned} & \ddot{x}_{1} - 2\left( {\alpha + \theta - \delta } \right)\dot{x}_{1} + \left[ {\left( {\alpha + \theta - \delta } \right)^{2} - \alpha^{2} } \right] = 0 \\ & y^{2} - 2\left( {\alpha + \theta - \delta } \right)y + \left[ {\left( {\alpha + \theta - \delta } \right)^{2} - \alpha^{2} } \right] = 0 \\ & y = \left( {\alpha + \theta - \delta } \right) \pm \alpha . \\ \end{aligned}$$

Therefore, \(x_{1} = c_{1} e^{{\left( {2\alpha + \theta - \delta } \right)t}} + c_{2} e^{{\left( {\theta - \delta } \right)t}} + N_{1}\).

Since \(x_{2} = \frac{1}{\alpha }\left[ {\dot{x}_{1} - \left( {\alpha + \theta - \delta } \right)x_{1} } \right],\,x_{2} = c_{1} e^{{\left( {2\alpha + \theta - \delta } \right)t}} - c_{2} e^{{\left( {\theta - \delta } \right)t}} + N_{2}\).

Now, a particular solution:

$$\begin{aligned} & 0 = \left( {\alpha + \theta - \delta } \right)x_{1} + \alpha x_{2} + \beta_{1} + \phi_{1} - \rho \\ & 0 = \alpha x_{1} + \left( {\alpha + \theta - \delta } \right)x_{2} + \beta_{2} + \phi_{2} - \rho \\ & x_{2} = \frac{{ - \alpha x_{1} - (\beta_{2} + \phi_{2} - \rho )}}{\alpha + \theta - \delta } \\ & \left( {\alpha + \theta - \delta } \right)x_{1} - \frac{{\alpha^{2} }}{\alpha + \theta - \delta }x_{1} - \frac{{\alpha \left( {\beta_{2} + \phi_{2} - \rho } \right)}}{\alpha + \theta - \delta } + \beta_{1} + \phi_{1} - \rho = 0 \\ & \left( {\alpha + \theta - \delta } \right)^{2} x_{1} - \alpha^{2} x_{1} - \alpha \left( {\beta_{2} + \phi_{2} - \rho } \right) + \left( {\alpha + \theta - \delta } \right)\left( {\beta_{1} + \phi_{1} - \rho } \right) = 0 \\ & x_{1} = \frac{{\alpha \left( {\beta_{2} + \phi_{2} - \rho } \right) - \left( {\alpha + \theta - \delta } \right)(\beta_{1} + \phi_{1} - \rho )}}{{\left( {\alpha + \theta - \delta } \right)^{2} - \alpha^{2} }} \\ & x_{2} = \frac{ - \alpha }{\alpha + \theta - \delta }\left( {\frac{{\alpha \left( {\beta_{2} + \phi_{2} - \rho } \right) - \left( {\alpha + \theta - \delta } \right)\left( {\beta_{1} + \phi_{1} - \rho } \right)}}{{\left( {\alpha + \theta - \delta } \right)^{2} - \alpha^{2} }}} \right) - \frac{{\beta_{2} + \phi_{2} - \rho }}{\alpha + \theta - \delta } \\ & x_{2} = \frac{{\alpha \left( {\beta_{1} + \phi_{1} - \rho } \right) - \left( {\alpha + \theta - \delta } \right)(\beta_{2} + \phi_{2} - \rho )}}{{\left( {\alpha + \theta - \delta } \right)^{2} - \alpha^{2} }} \\ \end{aligned}$$

Therefore,

$$\begin{aligned} x_{1} & = c_{1} e^{{\left( {2\alpha + \theta - \delta } \right)t}} + c_{2} e^{{\left( {\theta - \delta } \right)t}} + \frac{{\alpha \left( {\beta_{2} + \phi_{2} - \rho } \right) - \left( {\alpha + \theta - \delta } \right)(\beta_{1} + \phi_{1} - \rho )}}{{\left( {\alpha + \theta - \delta } \right)^{2} - \alpha^{2} }} \\ x_{2} & = c_{1} e^{{\left( {2\alpha + \theta - \delta } \right)t}} - c_{2} e^{{\left( {\theta - \delta } \right)t}} + \frac{{\alpha \left( {\beta_{1} + \phi_{1} - \rho } \right) - \left( {\alpha + \theta - \delta } \right)(\beta_{2} + \phi_{2} - \rho )}}{{\left( {\alpha + \theta - \delta } \right)^{2} - \alpha^{2} }} \\ \end{aligned}$$

To determine \(c_{1}\) and \(c_{2}\), assume \(x_{1} \left( 0 \right) = x_{10}\) and \(x_{2} \left( 0 \right) = {\text{x}}_{20}\).

(Further suppose \(x_{1} \left( 0 \right) = x_{2} \left( 0 \right) = 0\) to calculate \(c_{1}\) and \(c_{2}\).).

1.4 Derivation of Theorem 4

Now, consider a perfect coordination (i.e., collusive ) case:

$$\begin{aligned} J & = \mathop \smallint \limits_{0}^{\infty } e^{ - rt} \left\{ {\left( {x_{1} + x_{2} } \right)\left[ {a_{1} + a_{2} - 2\left( {x_{1} + x_{2} } \right)} \right] + s_{1} x_{1} \left[ {b_{1} - x_{1} } \right] + s_{2} x_{2} \left[ {b_{2} - x_{2} } \right]} \right. \\ & \quad \left. { - \rho \left( {u_{1} + u_{2} } \right) - \frac{1}{2}\left( {u_{1}^{2} + u_{2}^{2} } \right)} \right\}dt \\ \end{aligned}$$
$$\begin{aligned} \dot{x}_{1} & = u_{1} - \delta x_{1} \\ \dot{x}_{2} & = u_{2} - \delta x_{2} \\ \end{aligned}$$

Current value Hamiltonian is:

$$\begin{aligned} H & = \left( {x_{1} + x_{2} } \right)\left[ {a_{1} + a_{2} - 2\left( {x_{1} + x_{2} } \right)} \right] + s_{1} x_{1} \left[ {b_{1} - x_{1} } \right] + s_{2} x_{2} \left[ {b_{2} - x_{2} } \right] - \rho \left( {u_{1} + u_{2} } \right) \\ & \quad - \frac{1}{2}\left( {u_{1}^{2} + u_{2}^{2} } \right) + \lambda_{1} \left( {u_{1} - \delta x_{1} } \right) + \lambda_{2} \left( {u_{2} - \delta x_{2} } \right) \\ \end{aligned}$$

Necessary conditions :

$$\begin{aligned} & \frac{\partial H}{{\partial u_{1} }} = - \rho - u_{1} + \lambda_{1} = 0,\,u_{1} = \lambda_{1} - \rho ;\,\frac{\partial H}{{\partial u_{2} }} = - \rho - u_{2} + \lambda_{2} = 0,\,u_{2} = \lambda_{2} - \rho \\ & \dot{\lambda }_{1} = r\lambda_{1} - \frac{\partial H}{{\partial x_{1} }} = \left( {r + \delta } \right)\lambda_{1} + \left( {2s_{1} + 4} \right)x_{1} + 4x_{2} - (a_{1} + a_{2} + s_{1} b_{1} ). \\ \end{aligned}$$

Likewise, \(\dot{\lambda }_{2} = r\lambda_{2} - \frac{\partial H}{{\partial x_{2} }} = \left( {r + \delta } \right)\lambda_{2} + 4x_{1} + \left( {2s_{2} + 4} \right)x_{2} - (a_{1} + a_{2} + s_{2} b_{2} )\).

Now, we have

$$\begin{aligned} \dot{x}_{1} & = - \delta x_{1} + \lambda_{1} - \rho \\ \dot{x}_{2} & = - \delta x_{2} + \lambda_{2} - \rho \\ \dot{\lambda }_{1} & = \left( {2s_{1} + 4} \right)x_{1} + 4x_{2} + \left( {r + \delta } \right)\lambda_{1} - \left( {a_{1} + a_{2} + s_{1} b_{1} } \right) \\ \dot{\lambda }_{2} & = 4x_{1} + \left( {2s_{2} + 4} \right)x_{2} + \left( {r + \delta } \right)\lambda_{2} - \left( {a_{1} + a_{2} + s_{2} b_{2} } \right) \\ \end{aligned}$$

For homogeneous systems,

$$\left[ {\begin{array}{*{20}c} { - \delta } & 0 & 1 & 0 \\ 0 & { - \delta } & 0 & 1 \\ {2s_{1} + 4} & 4 & {r + \delta } & 0 \\ 4 & {2s_{2} + 4} & 0 & {r + \delta } \\ \end{array} } \right]\left[ {\begin{array}{*{20}c} {x_{1} } \\ {x_{2} } \\ {\lambda_{1} } \\ {\lambda_{2} } \\ \end{array} } \right]\left| {\begin{array}{*{20}c} { - \delta - z} & 0 & 1 & 0 \\ 0 & { - \delta - z} & 0 & 1 \\ {2s_{1} + 4} & 4 & {r + \delta - z} & 0 \\ 4 & {2s_{2} + 4} & 0 & {r + \delta - z} \\ \end{array} } \right| = 0$$
$$\left( { - \delta - z} \right)\left| {\begin{array}{*{20}c} { - \delta - z} & 0 & 1 \\ 4 & {r + \delta - z} & 0 \\ {2s_{2} + 4} & 0 & {r + \delta - z} \\ \end{array} } \right| + \left| {\begin{array}{*{20}c} 0 & { - \delta - z} & 1 \\ {2s_{1} + 4} & 4 & 0 \\ 4 & {2s_{2} + 4} & {r + \delta - z} \\ \end{array} } \right| = 0$$
$$\begin{aligned} & \left( { - \delta - z} \right)\left\{ {\left( { - \delta - z} \right)\left( {r + \delta - z} \right)^{2} - \left( {2s_{2} + 4} \right)\left( {r + \delta - z} \right)} \right\} + \left( {\delta + z} \right)\left( {2s_{1} + 4} \right)\left( {r + \delta - z} \right) \\ & + \left( {2s_{1} + 4} \right)\left( {2s_{2} + 4} \right) - 16 = 0 \\ \end{aligned}$$
$$\begin{aligned} & \left\{ {\left( {\delta + z} \right)\left( {r + \delta - z} \right)} \right\}^{2} + \left( {2s_{1} + 2s_{2} + 8} \right)\left\{ {\left( {\delta + z} \right)\left( {r + \delta - z} \right)} \right\} + \left( {2s_{1} + 4} \right)\left( {2s_{2} + 4} \right) - 16 = 0 \\ & \left( {\delta + z} \right)\left( {r + \delta - z} \right) = \frac{{ - \left( {2s_{1} + 2s_{2} + 8} \right) \pm \sqrt {\left( {2s_{1} + 2s_{2} + 8} \right)^{2} - 4\left\{ {\left( {2s_{1} + 4} \right)\left( {2s_{2} + 4} \right) - 16} \right\}} }}{2} \\ \end{aligned}$$

and we can determine 4 values of z.

For instance, suppose the case when \({\text{s}}_{1} = s_{2} = 1\).

$$\left[ {\left\{ {\left( {\delta + z} \right)\left( {r + \delta - z} \right)} \right\} + 2} \right]\left[ {\left\{ {\left( {\delta + z} \right)\left( {r + \delta - z} \right)} \right\} + 10} \right] = 0.$$

Therefore, \(\left( {\delta + z} \right)\left( {r + \delta - z} \right) = - 2\) or \(\left( {\delta + z} \right)\left( {r + \delta - z} \right) = - 10.\)

From \(\left( {\delta + z} \right)\left( {r + \delta - z} \right) = - 2\), we know \(z = \frac{{r \pm \sqrt {r^{2} + 4\left\{ {\delta \left( {r + \delta } \right) + 2} \right\}} }}{2}\).

From \(\left( {\delta + z} \right)\left( {r + \delta - z} \right) = - 10\), we know \(z = \frac{{r \pm \sqrt {r^{2} + 4\left\{ {\delta \left( {r + \delta } \right) + 10} \right\}} }}{2}\).

In order for the problem to have a solution, it must hold that \(\mathop {\lim }\limits_{t \to \infty } x_{i} \left( t \right)\) converges. Therefore, out of four possible values, we can have only two, i.e., \(z_{1} = \frac{{r - \sqrt {r^{2} + 4\left\{ {\delta \left( {r + \delta } \right) + 2} \right\}} }}{2}\) and \(z_{3} = \frac{{r - \sqrt {r^{2} + 4\left\{ {\delta \left( {r + \delta } \right) + 10} \right\}} }}{2}\) by posing \({\text{A}}_{2} = A_{4} = B_{2} = B_{4} = C_{2} = C_{4} = D_{2} = D_{4} = 0\).

Now, we have the following system.

$$\begin{aligned} x_{1} & = A_{1} e^{{z_{1} t}} + A_{3} e^{{z_{3} t}} + Constant \\ x_{2} & = B_{1} e^{{z_{1} t}} + B_{3} e^{{z_{3} t}} + Constant \\ \lambda_{1} & = C_{1} e^{{z_{1} t}} + C_{3} e^{{z_{3} t}} + Constant \\ \lambda_{2} & = D_{1} e^{{z_{1} t}} + D_{3} e^{{z_{3} t}} + Constant \\ \end{aligned}$$

In order to determine the constant terms, we can use the following set of relations.

$$0 = - \delta x_{1} + \lambda_{1} - \rho$$
(63)
$$0 = - \delta x_{2} + \lambda_{2} - \rho$$
(64)
$$\begin{aligned} 0 & = \left( {2s_{1} + 4} \right)x_{1} + 4x_{2} + \left( {r + \delta } \right)\lambda_{1} - \left( {a_{1} + a_{2} + s_{1} b_{1} } \right) \\ 0 & = 4x_{1} + \left( {2s_{2} + 4} \right)x_{2} + \left( {r + \delta } \right)\lambda_{2} - \left( {a_{1} + a_{2} + s_{2} b_{2} } \right) \\ \end{aligned}$$

After some arrangements, we have the following:

$$\begin{aligned} Const\left( {x_{1} } \right) & = \frac{{4\left[ {\rho \left( {r + \delta } \right) - \left( {a_{1} + a_{2} + s_{2} b_{2} } \right)} \right] - \left[ {\rho \left( {r + \delta } \right) - \left( {a_{1} + a_{2} + s_{1} b_{1} } \right)} \right]\left[ {\delta \left( {r + \delta } \right) + 2s_{2} + 4} \right]}}{{\left[ {\delta \left( {r + \delta } \right) + 2s_{1} + 4} \right]\left[ {\delta \left( {r + \delta } \right) + 2s_{2} + 4} \right] - 16}} \\ Const\left( {x_{2} } \right) & = \frac{{4\left[ {\rho \left( {r + \delta } \right) - \left( {a_{1} + a_{2} + s_{1} b_{1} } \right)} \right] - \left[ {\rho \left( {r + \delta } \right) - \left( {a_{1} + a_{2} + s_{2} b_{2} } \right)} \right]\left[ {\delta \left( {r + \delta } \right) + 2s_{1} + 4} \right]}}{{\left[ {\delta \left( {r + \delta } \right) + 2s_{1} + 4} \right]\left[ {\delta \left( {r + \delta } \right) + 2s_{2} + 4} \right] - 16}}. \\ \end{aligned}$$

Now we have:

$$\begin{aligned} x_{1} & = A_{1} e^{{z_{1} t}} + A_{3} e^{{z_{3} t}} + Const(x_{1} ) \\ x_{2} & = B_{1} e^{{z_{1} t}} + B_{3} e^{{z_{3} t}} + Const(x_{2} ) \\ \end{aligned}$$

From (63) and (64), we know:

$$\begin{aligned} & Const\left( {\lambda_{1} } \right) = \delta Const\left( {x_{1} } \right) + \rho \,{\text{and}}\,Const\left( {\lambda_{2} } \right) = \delta Const\left( {x_{2} } \right) + \rho . \\ & \lambda_{1} = C_{1} e^{{z_{1} t}} + C_{3} e^{{z_{3} t}} + Const(\lambda_{1} ) \\ & \lambda_{2} = D_{1} e^{{z_{1} t}} + D_{3} e^{{z_{3} t}} + Const(\lambda_{2} ) \\ \end{aligned}$$

Now, how to determine \(A_{1} ,\,A_{3} ,\,B_{1} ,\,B_{3} ,\,C_{1} ,\,C_{3} ,\,D_{1} ,\,D_{3}\) ?

Since \(\dot{x}_{1} = - \delta x_{1} + \lambda_{1} - \rho\) and \(\dot{x}_{2} = - \delta x_{2} + \lambda_{2} - \rho\),

$$\begin{aligned} & A_{1} z_{1} e^{{z_{1} t}} + A_{3} z_{3} e^{{z_{3} t}} = - \delta A_{1} e^{{z_{1} t}} - \delta A_{3} e^{{z_{3} t}} - \delta Const\left( {x_{1} } \right) + C_{1} e^{{z_{1} t}} + C_{3} e^{{z_{3} t}} \\ & + Const\left( {\lambda_{1} } \right) - \rho . \\ \end{aligned}$$
$$A_{1} \left( {z_{1} + \delta } \right) = C_{1} ,\,A_{3} \left( {z_{3} + \delta } \right) = C_{3} ,\,\delta Const\left( {x_{1} } \right) - Const\left( {\lambda_{1} } \right) + \rho = 0.$$

Likewise, \(B_{1} \left( {z_{1} + \delta } \right) = D_{1} ,\,D_{3} \left( {z_{3} + \delta } \right) = D_{3}\).

Therefore, we only need to determine \(A_{1} ,A_{3} ,B_{1} ,B_{3}\). It will be possible to determine exact values of these parameters, if specific initial conditions for the state variables are provided. For our purpose, we only need the steady-state values, which are represented by the constants in the state variables.

1.5 Derivation of Observation 1

\(X_{1N} \le X_{1C}\) implies \(\frac{{2\rho \left( {r + \delta - \alpha } \right) - \left( {a_{1} + a_{2} } \right)}}{{\left( {r + \delta - 3\alpha } \right)\left( {2\alpha - \delta } \right)}} \le \frac{{2\left[ {\left( {a_{1} + a_{2} } \right) - \rho \left( {r + \delta } \right)} \right]}}{{\delta \left( {r + \delta } \right) + 8}}\).

That is, \(\because \left( {r + \delta - 3\alpha } \right)\left( {2\alpha - \delta } \right) < 0\),

$$\left( {\delta \left( {r + \delta } \right) + 8} \right)\left[ {2\rho \left( {r + \delta - \alpha } \right) - \left( {a_{1} + a_{2} } \right)} \right] \ge 2\left[ {\left( {a_{1} + a_{2} } \right) - \rho \left( {r + \delta } \right)} \right]\left( {r + \delta - 3\alpha } \right)\left( {2\alpha - \delta } \right)$$
(65)

Now,

$$\begin{aligned} RHS & = 2\left[ {\left( {a_{1} + a_{2} } \right) - \rho \left( {r + \delta } \right)} \right]\left( {r + \delta - 3\alpha } \right)\left( {2\alpha - \delta } \right) \\ & = 2\left[ {\left( {a_{1} + a_{2} } \right) - \rho \left( {r + \delta } \right)} \right]\left[ { - 6\alpha^{2} + \alpha \left( {2r + 5\delta } \right) - \delta \left( {r + \delta } \right)} \right] \\ \end{aligned}$$

and \(3\alpha^{2} = \left( {r + 2\delta } \right)\alpha + 2\) (see p. 261 in Dockner et al. (2000)).

$$\therefore\, RHS = 2\left[ {\left( {a_{1} + a_{2} } \right) - \rho \left( {r + \delta } \right)} \right]\left( {\delta \alpha - 4 - \delta \left( {r + \delta } \right)} \right)$$

Then, from (65)

$$\begin{aligned} & \left( {\delta \left( {r + \delta } \right) + 8} \right)\left[ {2\rho \left( {r + \delta } \right) - \left( {a_{1} + a_{2} } \right)} \right] - 2\alpha \rho \left( {\delta \left( {r + \delta } \right) + 8} \right) \\ & \ge 2\left[ {\left( {a_{1} + a_{2} } \right) - \rho \left( {r + \delta } \right)} \right]\left( {\delta \alpha - 4 - \delta \left( {r + \delta } \right)} \right) \\ & \left( {\delta \left( {r + \delta } \right) + 8} \right)\left[ {2\rho \left( {r + \delta } \right) - \left( {a_{1} + a_{2} } \right)} \right] + 2\left[ {\left( {a_{1} + a_{2} } \right) - \rho \left( {r + \delta } \right)} \right]\delta \left( {r + \delta } \right) \ge 2[(a_{1} \\ & + a_{2} ) - [(\rho (r + \delta )](\delta \alpha - 4) + 2\alpha \rho (\delta (r + \delta ) + 8) \\ & \delta \left( {r + \delta } \right)\left( {a_{1} + a_{2} } \right) + 16\rho \left( {r + \delta } \right) - 8\left( {a_{1} + a_{2} } \right) \\ & \ge 2\delta \alpha \left( {a_{1} + a_{2} } \right) - 8\left( {a_{1} + a_{2} } \right) + 8\rho \left( {r + \delta } \right) + 16\alpha \rho \\ & \delta \left( {r + \delta } \right)\left( {a_{1} + a_{2} } \right) + 8\rho \left( {r + \delta } \right) \ge 2\alpha \left\{ {8\rho + \delta \left( {a_{1} + a_{2} } \right)} \right\} \\ & \left( {r + \delta } \right)\left\{ {8\rho + \delta \left( {a_{1} + a_{2} } \right)} \right\} \ge 2\alpha \left\{ {8\rho + \delta \left( {a_{1} + a_{2} } \right)} \right\} \\ \end{aligned}$$

Therefore, \(X_{1N} \le X_{1C}\) if and only if \(\frac{(r + \delta )}{2} \ge \alpha\).

Since α is negative for all r and δ, \(\frac{(r + \delta )}{2} \ge \alpha\) holds always and so does \(X_{1N} \le X_{1C}\).

Appendix 3

See Appendix Tables 6 and 7.

Rights and permissions

Reprints and permissions

Copyright information

© 2017 Springer Nature Singapore Pte Ltd.

About this chapter

Cite this chapter

Kim, B. (2017). Innovation Competition and Strategy. In: Optimal Control Applications for Operations Strategy. Springer, Singapore. https://doi.org/10.1007/978-981-10-3599-9_3

Download citation

Publish with us

Policies and ethics